LSAT and Law School Admissions Forum

Get expert LSAT preparation and law school admissions advice from PowerScore Test Preparation.

 Robert Carroll
PowerScore Staff
  • PowerScore Staff
  • Posts: 1787
  • Joined: Dec 06, 2013
|
#73370
Shannon,

Note the actual wording of answer choice (D): "could conceivably". The author of passage A does not have to be committed to any actual benefits for those benefits to be conceivable. Looking around line 25, I can also see that the author thinks the right thing is not justified merely when it produces good outcomes - thus, the author is certainly conceiving of a situation where there's a right thing to do and yet it doesn't produce a good outcome, so that the normative argument has to be marshaled to prove that it's still the right thing.

Robert Carroll
 frk215
  • Posts: 33
  • Joined: Sep 07, 2020
|
#95221
You put it into words PERFECTLY. I hate it. I ended up selecting e because even that felt like a better option, though i could clearly see why it would be wrong.
ShannonOh22 wrote:To phrase the question as "each author implies" is particularly insidious, even for the writers of the LSAT.

There is no part of passage A in which the author actually implies any conceivable benefits of lack of judicial candor...all author A does is neglect to disagree with the the proponents of prudential reasoning.

The fact that he starts his sentence with "If"(ln.3) isn't meant to be read as a possible concession or implication to the contrary - it follows directly from the preceding premise "The first [way to defend the principle] is to marshal prudential reasons that support the principle." There could have been a colon used here, as it seems the author is quoting one of the reasons.

Still not convinced that D is defensible at all...but I don't make the rules, so I will retreat in a silent rage and just try to remember these questions are not developed for the faint of heart.
 BMM2021
  • Posts: 39
  • Joined: Jun 30, 2021
|
#95580
Hi,

While I understand where the reasoning is coming from for Answer D, what's wrong with answer E? Passage B explicitly states this idea, of course, but I found myself using the sentence in passage A that states that greater candor could strengthen the institutional legitimacy of the courts to support the idea in answer E as well. I know it's sort of logically weak to assume that the stated concept (more candor = more legitimacy) could be reversed (less candor = less legitimacy), but if that assumption holds, it follows that a lack of candor resulting in less legitimacy means that the lack of candor was ultimately detected by those who afford the courts their legitimacy.

Ultimately, I feel like that "jump" is just as far as saying that passage A actually supports the idea that a lack of candor could be beneficial. How does one choose between inferences?
 Adam Tyson
PowerScore Staff
  • PowerScore Staff
  • Posts: 5153
  • Joined: Apr 14, 2011
|
#96357
Actually, author B does NOT say that a lack of candor is likely to be detected. They say it is likely to be detectable, and that is not the same thing, and we should not make any assumptions that something detectable will eventually be detected. To see a parallel case, there are a lot of things in my home right now that are recyclable, but that doesn't mean any of them will, eventually, be recycled. Maybe I'll just throw them in the trash and they will all end up in a landfill?

And author A, of course, says nothing at all like this, which is even more reason to eliminate this answer.

Get the most out of your LSAT Prep Plus subscription.

Analyze and track your performance with our Testing and Analytics Package.